8
$\begingroup$

For $n\in \mathbb{N}$ with prime decomposition $n=p_1^{r_1}\cdots p_k^{r_k},p_i\neq p_j$, let $A=\{p_1,\cdots,p_k\}$; then the following holds: \begin{equation} |\{q\in \mathbb{N},q<Q: \text{all prime factors of } q\,\in A\}|<Ce^{c\frac{\log Q}{\log \log Q}}d(n,Q) \end{equation} where $d(n,Q):=|\{m\in \mathbb{N},m<Q: m\mid n\}|$.

This lemma appears below (3.46) in Bourgain's article "Fourier transform restriction phenomena for certain lattice subsets and applications to nonlinear evolution equations".

I'm not familiar with number theory; could you please explain how to deduce this conclusion and give some references on number theory?

$\endgroup$

2 Answers 2

10
$\begingroup$

Let $q<Q$ be such that all its prime divisors are in $A$. We can write $q=q_0\times r_1^{n_1}\cdots r_s^{n_s}$, where all prime factors of $q_0$ are $<\log Q$ and $\log Q\leq r_1<\cdots<r_s\in A$. Clearly, $q$ is uniquely determined once we have fixed a) $q_0$, b) the set $\{r_1,\ldots,r_s\}$, and c) the integer vector $(n_1,\ldots,n_s)$.

The number of possible $q_0$ is at most the number of $q<Q$ which have all prime factors $<\log Q$. This is a well-studied quantity in analytic number theory (counting 'smooth' or 'friable' numbers), and is $\leq \exp(O(\log Q/\log\log Q))$, which can be shown via elementary methods, see any textbook on analytic number theory.

There are clearly at most $d(n,Q)$ choices for the set $\{r_1,\ldots,r_k\}$.

Finally, to estimate the number of choices for (c), note that $$\sum n_i\log r_i < \log Q,$$ and $\log r_i\gg \log \log Q$ by construction, hence it suffices to bound the number of positive integers $n_1,\ldots,n_s\geq 1$ (where $s$ may vary) such that $\sum n_i \ll \log Q/\log\log Q$. This is at most $\exp(O(\log Q/\log\log Q))$, using the elementary fact (simple combinatorics, 'stars and bars') that the the number of sequences of positive integers $n_i$ with $\sum n_i \leq M$ is exactly $2^M$.

Thus combining these three estimates, the number of possible $q$ is at most

$$ (\exp(O(\log Q/\log\log Q)))^2\times d(n,Q) \ll \exp(O(\log Q/\log\log Q))d(n,Q).$$

$\endgroup$
1
  • $\begingroup$ Thank you! By restricting all prime factors of $q_0<C_{\epsilon} (\text{rather than }\log Q)$, we can get a bound $Q^{\epsilon}d(n,Q)$, which is sufficient for the article. $\endgroup$ Mar 25, 2022 at 1:00
6
$\begingroup$

Here is a proof that its at most $\exp(O(\log Q\log\log\log Q/\log \log Q))d(n,Q)$, which suffices for the paper.

Let $S(n,Q) = \{q<Q: q|n\}$. Clearly $|S(n,Q)| = d(n,Q)$.

Now consider $q<Q$ with prime factors in $A$. We define $q'=f(q)$ to be the maximal $q' \in S(n,Q)$ such that $q'|q$ (clearly $q'$ is well-defined as $q<Q$).

We now claim that $|f^{-1}(q')| \le \exp(O(\log Q/\log\log Q))$ for each $q'\in S(n,Q)$. Let $A'$ be the primes dividing $q'$. Since $$Q> q' \ge \prod_{p \in A'}p,$$ we have $|A'| := k' \le (1+o(1))\log Q/\log\log Q$ by PNT.

We next observe that $|f^{-1}(q')|$ is upper bounded by the number of $q<Q$ with prime factors in $A'$. Letting $P_1,\dots,P_{k'}$ be the $k'$ smallest primes, the number of such $q$ is bounded by the number of $i_1,\dots,i_{k'}\ge 1$ such that$$\sum_j i_j \log P_j < \log Q.$$

But this at most the volume of a $k'$-dimensional unit simplex with the $j$-th axis scaled by $2\log Q/\log P_j$. The volume of the unit simplex is $1/k'!$ and the determinant of our scaling is $$D =\exp(k'(\log(2)+\log \log Q) - \sum_j \log\log P_j)\le \exp(k' +k'\log\log Q) .$$ If $k'<\log Q/(\log\log Q)^2$ we are done since $1/k'!<1$. Otherwise, we have $1/k'!\le \exp(-k'(\log k'-2))$ for all large $k'$ (Stirling's approximation), so here $D/k'! \le \exp(3k' + k'(\log\log Q-\log k')) $. Since $\log\log Q -\log k'\le 2\log\log \log Q$ (for our range of $k'$ we are done.

$\endgroup$
2
  • 1
    $\begingroup$ Thank you! There might be a typo in the definition of $S(n,Q)$. $\endgroup$ Mar 25, 2022 at 1:03
  • 1
    $\begingroup$ indeed, it is fixed. $\endgroup$ Mar 25, 2022 at 3:16

Your Answer

By clicking “Post Your Answer”, you agree to our terms of service and acknowledge that you have read and understand our privacy policy and code of conduct.

Not the answer you're looking for? Browse other questions tagged or ask your own question.